Convergence for improper integral $int_0^infty x^re^{-x} dx$












3












$begingroup$


I'm trying to find for which values $r$ the following improper integral converges.
$$int_0^infty x^re^{-x} dx$$
What I have so far is that $x^r < e^{frac{1}{2}x}$ for $x geq a$, which splits the integral into
$$int_0^a x^re^{-x} dx + int_a^infty e^{-frac{1}{2}x}$$
We know the latter interval converges, but I don't know what to do with the first one. (For reference, graphing the functions reveals the answer to be $x > -1$.)



Edit: I would like a proof without the gamma function. Preferably one that uses the comparison test to compare limits.










share|cite|improve this question











$endgroup$








  • 6




    $begingroup$
    This is actually one of the more interesting integrals in mathematics. try plugging in $ r = 1, 2, 3, 4...$ etc. What do you notice?
    $endgroup$
    – Aniruddh Venkatesan
    Jan 18 at 3:41






  • 1




    $begingroup$
    Hint:$$intlimits_0^{infty}mathrm dz, x^n e^{-x}=n!=Gamma(n+1)$$
    $endgroup$
    – Frank W.
    Jan 18 at 3:42






  • 3




    $begingroup$
    Did anybody even read the OP's question or do they just want to use the post to mention the gamma function?
    $endgroup$
    – Dionel Jaime
    Jan 18 at 3:49








  • 1




    $begingroup$
    @AniruddhVenkatesan I know it equals $n!$ for natural numbers, but for this question I'm mainly interested in for which real values of $r$ the interval converges.
    $endgroup$
    – Chase K
    Jan 18 at 3:53










  • $begingroup$
    @DionelJaime As I recall, the original question didn't mention not using the gamma function, with this being added afterwards.
    $endgroup$
    – John Omielan
    Jan 18 at 3:58
















3












$begingroup$


I'm trying to find for which values $r$ the following improper integral converges.
$$int_0^infty x^re^{-x} dx$$
What I have so far is that $x^r < e^{frac{1}{2}x}$ for $x geq a$, which splits the integral into
$$int_0^a x^re^{-x} dx + int_a^infty e^{-frac{1}{2}x}$$
We know the latter interval converges, but I don't know what to do with the first one. (For reference, graphing the functions reveals the answer to be $x > -1$.)



Edit: I would like a proof without the gamma function. Preferably one that uses the comparison test to compare limits.










share|cite|improve this question











$endgroup$








  • 6




    $begingroup$
    This is actually one of the more interesting integrals in mathematics. try plugging in $ r = 1, 2, 3, 4...$ etc. What do you notice?
    $endgroup$
    – Aniruddh Venkatesan
    Jan 18 at 3:41






  • 1




    $begingroup$
    Hint:$$intlimits_0^{infty}mathrm dz, x^n e^{-x}=n!=Gamma(n+1)$$
    $endgroup$
    – Frank W.
    Jan 18 at 3:42






  • 3




    $begingroup$
    Did anybody even read the OP's question or do they just want to use the post to mention the gamma function?
    $endgroup$
    – Dionel Jaime
    Jan 18 at 3:49








  • 1




    $begingroup$
    @AniruddhVenkatesan I know it equals $n!$ for natural numbers, but for this question I'm mainly interested in for which real values of $r$ the interval converges.
    $endgroup$
    – Chase K
    Jan 18 at 3:53










  • $begingroup$
    @DionelJaime As I recall, the original question didn't mention not using the gamma function, with this being added afterwards.
    $endgroup$
    – John Omielan
    Jan 18 at 3:58














3












3








3





$begingroup$


I'm trying to find for which values $r$ the following improper integral converges.
$$int_0^infty x^re^{-x} dx$$
What I have so far is that $x^r < e^{frac{1}{2}x}$ for $x geq a$, which splits the integral into
$$int_0^a x^re^{-x} dx + int_a^infty e^{-frac{1}{2}x}$$
We know the latter interval converges, but I don't know what to do with the first one. (For reference, graphing the functions reveals the answer to be $x > -1$.)



Edit: I would like a proof without the gamma function. Preferably one that uses the comparison test to compare limits.










share|cite|improve this question











$endgroup$




I'm trying to find for which values $r$ the following improper integral converges.
$$int_0^infty x^re^{-x} dx$$
What I have so far is that $x^r < e^{frac{1}{2}x}$ for $x geq a$, which splits the integral into
$$int_0^a x^re^{-x} dx + int_a^infty e^{-frac{1}{2}x}$$
We know the latter interval converges, but I don't know what to do with the first one. (For reference, graphing the functions reveals the answer to be $x > -1$.)



Edit: I would like a proof without the gamma function. Preferably one that uses the comparison test to compare limits.







calculus integration improper-integrals






share|cite|improve this question















share|cite|improve this question













share|cite|improve this question




share|cite|improve this question








edited Jan 18 at 3:55







Chase K

















asked Jan 18 at 3:38









Chase KChase K

675




675








  • 6




    $begingroup$
    This is actually one of the more interesting integrals in mathematics. try plugging in $ r = 1, 2, 3, 4...$ etc. What do you notice?
    $endgroup$
    – Aniruddh Venkatesan
    Jan 18 at 3:41






  • 1




    $begingroup$
    Hint:$$intlimits_0^{infty}mathrm dz, x^n e^{-x}=n!=Gamma(n+1)$$
    $endgroup$
    – Frank W.
    Jan 18 at 3:42






  • 3




    $begingroup$
    Did anybody even read the OP's question or do they just want to use the post to mention the gamma function?
    $endgroup$
    – Dionel Jaime
    Jan 18 at 3:49








  • 1




    $begingroup$
    @AniruddhVenkatesan I know it equals $n!$ for natural numbers, but for this question I'm mainly interested in for which real values of $r$ the interval converges.
    $endgroup$
    – Chase K
    Jan 18 at 3:53










  • $begingroup$
    @DionelJaime As I recall, the original question didn't mention not using the gamma function, with this being added afterwards.
    $endgroup$
    – John Omielan
    Jan 18 at 3:58














  • 6




    $begingroup$
    This is actually one of the more interesting integrals in mathematics. try plugging in $ r = 1, 2, 3, 4...$ etc. What do you notice?
    $endgroup$
    – Aniruddh Venkatesan
    Jan 18 at 3:41






  • 1




    $begingroup$
    Hint:$$intlimits_0^{infty}mathrm dz, x^n e^{-x}=n!=Gamma(n+1)$$
    $endgroup$
    – Frank W.
    Jan 18 at 3:42






  • 3




    $begingroup$
    Did anybody even read the OP's question or do they just want to use the post to mention the gamma function?
    $endgroup$
    – Dionel Jaime
    Jan 18 at 3:49








  • 1




    $begingroup$
    @AniruddhVenkatesan I know it equals $n!$ for natural numbers, but for this question I'm mainly interested in for which real values of $r$ the interval converges.
    $endgroup$
    – Chase K
    Jan 18 at 3:53










  • $begingroup$
    @DionelJaime As I recall, the original question didn't mention not using the gamma function, with this being added afterwards.
    $endgroup$
    – John Omielan
    Jan 18 at 3:58








6




6




$begingroup$
This is actually one of the more interesting integrals in mathematics. try plugging in $ r = 1, 2, 3, 4...$ etc. What do you notice?
$endgroup$
– Aniruddh Venkatesan
Jan 18 at 3:41




$begingroup$
This is actually one of the more interesting integrals in mathematics. try plugging in $ r = 1, 2, 3, 4...$ etc. What do you notice?
$endgroup$
– Aniruddh Venkatesan
Jan 18 at 3:41




1




1




$begingroup$
Hint:$$intlimits_0^{infty}mathrm dz, x^n e^{-x}=n!=Gamma(n+1)$$
$endgroup$
– Frank W.
Jan 18 at 3:42




$begingroup$
Hint:$$intlimits_0^{infty}mathrm dz, x^n e^{-x}=n!=Gamma(n+1)$$
$endgroup$
– Frank W.
Jan 18 at 3:42




3




3




$begingroup$
Did anybody even read the OP's question or do they just want to use the post to mention the gamma function?
$endgroup$
– Dionel Jaime
Jan 18 at 3:49






$begingroup$
Did anybody even read the OP's question or do they just want to use the post to mention the gamma function?
$endgroup$
– Dionel Jaime
Jan 18 at 3:49






1




1




$begingroup$
@AniruddhVenkatesan I know it equals $n!$ for natural numbers, but for this question I'm mainly interested in for which real values of $r$ the interval converges.
$endgroup$
– Chase K
Jan 18 at 3:53




$begingroup$
@AniruddhVenkatesan I know it equals $n!$ for natural numbers, but for this question I'm mainly interested in for which real values of $r$ the interval converges.
$endgroup$
– Chase K
Jan 18 at 3:53












$begingroup$
@DionelJaime As I recall, the original question didn't mention not using the gamma function, with this being added afterwards.
$endgroup$
– John Omielan
Jan 18 at 3:58




$begingroup$
@DionelJaime As I recall, the original question didn't mention not using the gamma function, with this being added afterwards.
$endgroup$
– John Omielan
Jan 18 at 3:58










1 Answer
1






active

oldest

votes


















6












$begingroup$

Note that for all $r in mathbb{R}$,



$$lim_{x to infty} frac{x^r e^{-x}}{x^{-2}} = lim_{x to infty} frac{x^{r+2}}{e^x} = 0 $$



since the exponential function tends to infinity faster than any polynomial. Hence, by the limit comparison test the integral over $[1, infty)$ converges since $int_1^infty x^{-2}, dx = 1$.



See if you can finish by finding the condition on $r$ such that the integral over $[0,1]$ converges.






share|cite|improve this answer









$endgroup$













    Your Answer





    StackExchange.ifUsing("editor", function () {
    return StackExchange.using("mathjaxEditing", function () {
    StackExchange.MarkdownEditor.creationCallbacks.add(function (editor, postfix) {
    StackExchange.mathjaxEditing.prepareWmdForMathJax(editor, postfix, [["$", "$"], ["\\(","\\)"]]);
    });
    });
    }, "mathjax-editing");

    StackExchange.ready(function() {
    var channelOptions = {
    tags: "".split(" "),
    id: "69"
    };
    initTagRenderer("".split(" "), "".split(" "), channelOptions);

    StackExchange.using("externalEditor", function() {
    // Have to fire editor after snippets, if snippets enabled
    if (StackExchange.settings.snippets.snippetsEnabled) {
    StackExchange.using("snippets", function() {
    createEditor();
    });
    }
    else {
    createEditor();
    }
    });

    function createEditor() {
    StackExchange.prepareEditor({
    heartbeatType: 'answer',
    autoActivateHeartbeat: false,
    convertImagesToLinks: true,
    noModals: true,
    showLowRepImageUploadWarning: true,
    reputationToPostImages: 10,
    bindNavPrevention: true,
    postfix: "",
    imageUploader: {
    brandingHtml: "Powered by u003ca class="icon-imgur-white" href="https://imgur.com/"u003eu003c/au003e",
    contentPolicyHtml: "User contributions licensed under u003ca href="https://creativecommons.org/licenses/by-sa/3.0/"u003ecc by-sa 3.0 with attribution requiredu003c/au003e u003ca href="https://stackoverflow.com/legal/content-policy"u003e(content policy)u003c/au003e",
    allowUrls: true
    },
    noCode: true, onDemand: true,
    discardSelector: ".discard-answer"
    ,immediatelyShowMarkdownHelp:true
    });


    }
    });














    draft saved

    draft discarded


















    StackExchange.ready(
    function () {
    StackExchange.openid.initPostLogin('.new-post-login', 'https%3a%2f%2fmath.stackexchange.com%2fquestions%2f3077808%2fconvergence-for-improper-integral-int-0-infty-xre-x-dx%23new-answer', 'question_page');
    }
    );

    Post as a guest















    Required, but never shown

























    1 Answer
    1






    active

    oldest

    votes








    1 Answer
    1






    active

    oldest

    votes









    active

    oldest

    votes






    active

    oldest

    votes









    6












    $begingroup$

    Note that for all $r in mathbb{R}$,



    $$lim_{x to infty} frac{x^r e^{-x}}{x^{-2}} = lim_{x to infty} frac{x^{r+2}}{e^x} = 0 $$



    since the exponential function tends to infinity faster than any polynomial. Hence, by the limit comparison test the integral over $[1, infty)$ converges since $int_1^infty x^{-2}, dx = 1$.



    See if you can finish by finding the condition on $r$ such that the integral over $[0,1]$ converges.






    share|cite|improve this answer









    $endgroup$


















      6












      $begingroup$

      Note that for all $r in mathbb{R}$,



      $$lim_{x to infty} frac{x^r e^{-x}}{x^{-2}} = lim_{x to infty} frac{x^{r+2}}{e^x} = 0 $$



      since the exponential function tends to infinity faster than any polynomial. Hence, by the limit comparison test the integral over $[1, infty)$ converges since $int_1^infty x^{-2}, dx = 1$.



      See if you can finish by finding the condition on $r$ such that the integral over $[0,1]$ converges.






      share|cite|improve this answer









      $endgroup$
















        6












        6








        6





        $begingroup$

        Note that for all $r in mathbb{R}$,



        $$lim_{x to infty} frac{x^r e^{-x}}{x^{-2}} = lim_{x to infty} frac{x^{r+2}}{e^x} = 0 $$



        since the exponential function tends to infinity faster than any polynomial. Hence, by the limit comparison test the integral over $[1, infty)$ converges since $int_1^infty x^{-2}, dx = 1$.



        See if you can finish by finding the condition on $r$ such that the integral over $[0,1]$ converges.






        share|cite|improve this answer









        $endgroup$



        Note that for all $r in mathbb{R}$,



        $$lim_{x to infty} frac{x^r e^{-x}}{x^{-2}} = lim_{x to infty} frac{x^{r+2}}{e^x} = 0 $$



        since the exponential function tends to infinity faster than any polynomial. Hence, by the limit comparison test the integral over $[1, infty)$ converges since $int_1^infty x^{-2}, dx = 1$.



        See if you can finish by finding the condition on $r$ such that the integral over $[0,1]$ converges.







        share|cite|improve this answer












        share|cite|improve this answer



        share|cite|improve this answer










        answered Jan 18 at 4:33









        RRLRRL

        51.1k42573




        51.1k42573






























            draft saved

            draft discarded




















































            Thanks for contributing an answer to Mathematics Stack Exchange!


            • Please be sure to answer the question. Provide details and share your research!

            But avoid



            • Asking for help, clarification, or responding to other answers.

            • Making statements based on opinion; back them up with references or personal experience.


            Use MathJax to format equations. MathJax reference.


            To learn more, see our tips on writing great answers.




            draft saved


            draft discarded














            StackExchange.ready(
            function () {
            StackExchange.openid.initPostLogin('.new-post-login', 'https%3a%2f%2fmath.stackexchange.com%2fquestions%2f3077808%2fconvergence-for-improper-integral-int-0-infty-xre-x-dx%23new-answer', 'question_page');
            }
            );

            Post as a guest















            Required, but never shown





















































            Required, but never shown














            Required, but never shown












            Required, but never shown







            Required, but never shown

































            Required, but never shown














            Required, but never shown












            Required, but never shown







            Required, but never shown







            Popular posts from this blog

            Mario Kart Wii

            What does “Dominus providebit” mean?

            Antonio Litta Visconti Arese